Differential Equations - Logistic Model

Click For Summary
The discussion centers on solving the logistic differential equation dP/dt = kP(1 - P/A), where k is a constant, A is the carrying capacity, and P is the population over time. The user successfully derived the integration constant c but struggles to match their solution for P(t) with the form presented in their calculus book. They detail their integration process and express confusion about the transition to the book's solution, P(t) = 300/(1 + 5e-0.1t). Clarification is sought on the steps needed to derive P(t) correctly from the general equation. The conversation emphasizes the importance of proper integration and manipulation of the equation to arrive at the correct logistic model solution.
NSOutWest
Messages
4
Reaction score
0
I have the equation dP/dt = kP(1 - P/A). It is supposed to describe a logistical situatuon involving the carrying capacity of the system.

k is a constant, and A is the carrying capacity of the system. t is time and P is population as a function of time. P(0) = P0. I solved c (the integration constant) to be:

c = -ln|(P0)/(A - P0)|

I'm trying to solve the equation in terms of t.

In my calculus book, a similar equation is given with an explanation.

dP/dt = 0.1P(1 - P/300)

With an initial condition of P(0) = 50, c is found to be ln(1/5). A = 300 and k = 0.1.

I follow along well up to this point.

After solving for c, the book lists the rearranged equation as:

P(t) = 300/(1 + 5e-0.1t)

I don't understand how they went from one equation to the other, the closest I could come with the general equation was:

P(t) = (Aekt + Q)/(1 + ekt) where Q = (P0)/(A - P0)

Which would coincide with an equation of:

(300e0.1t + .2)/(1 + e0.1t)

Which, when graphed, is not equivalent to the equation given by the book.

Can anyone go over how to solve the general equation? I think I'm missing some crucial point.

Thank you!
 
Last edited:
Physics news on Phys.org
This equation is separable - meaning that you can get all of the terms involving P on one side (including dP), all of the terms involving t on the other (including dt), and integrate both sides.

So we write the equation as

\left(\frac{1}{P} + \frac{1/A}{1-P/A}\right)\ dP = k\ dt

Integrate both sides to get ln|P| - ln|1-P/A| = kt + c, take the exponential of both sides, and you'll get your answer
 
That's what I've done. I'm trying to solve for P, and I can't figure out where my method diverges from the book's explanation.

I was hoping someone could show me the steps to solving for P, so I can figure out where my error lies.
 
Well, exp(ln|P|-ln|1-P/A|) = exp(kt + c)

The right hand side simplifies to exp(kt+c) = exp(kt)*exp(c) = C exp(kt), where C is a constant that we'll determine later.

The left hand side simplifies to exp(ln|P|-ln|1-P/A|) = exp(ln|P|) exp(-ln|1-P/A|) = P/(1-P/A)

So P/(1-P/A) =C exp(kt)
P = (A-P) C exp(kt)/A
P = AC exp(kt) - (PC/A)exp(kt)
P + (PC/A) exp(kt) = AC exp(kt)
P(1 + C/A exp(kt)) = AC exp(kt)
P = AC exp(kt)/(1+C/A exp(kt))

Multiplying the numerator and denominator of the RHS by exp(-kt)

P = AC/(exp(-kt) + C/A)
 
Question: A clock's minute hand has length 4 and its hour hand has length 3. What is the distance between the tips at the moment when it is increasing most rapidly?(Putnam Exam Question) Answer: Making assumption that both the hands moves at constant angular velocities, the answer is ## \sqrt{7} .## But don't you think this assumption is somewhat doubtful and wrong?

Similar threads

  • · Replies 7 ·
Replies
7
Views
6K
  • · Replies 7 ·
Replies
7
Views
2K
  • · Replies 1 ·
Replies
1
Views
1K
  • · Replies 3 ·
Replies
3
Views
1K
  • · Replies 5 ·
Replies
5
Views
2K
  • · Replies 4 ·
Replies
4
Views
2K
  • · Replies 5 ·
Replies
5
Views
2K
  • · Replies 7 ·
Replies
7
Views
1K
  • · Replies 2 ·
Replies
2
Views
1K
  • · Replies 7 ·
Replies
7
Views
1K